- PowerScore Staff
- Posts: 5972
- Joined: Mar 25, 2011
- Fri Mar 12, 2021 3:27 pm
#85361
Complete Question Explanation
(The complete setup for this game can be found here: lsat/viewtopic.php?f=315&t=9170)
The correct answer choice is (E)
Answer choice (A) is incorrect because from the second rule L cannot view the site on day 6.
Answer choice (B) is incorrect because from the first rule F must view the site on day 3 or day 5, and thus cannot view the site on day 4.
Answer choice (C) is incorrect because from the last rule G must view the site on the day after H views the site.
Answer choice (D) is incorrect because from the fourth rule, when K views the site on day 4, L must view the site on day 5.
Answer choice (E) is the correct answer.
(The complete setup for this game can be found here: lsat/viewtopic.php?f=315&t=9170)
The correct answer choice is (E)
Answer choice (A) is incorrect because from the second rule L cannot view the site on day 6.
Answer choice (B) is incorrect because from the first rule F must view the site on day 3 or day 5, and thus cannot view the site on day 4.
Answer choice (C) is incorrect because from the last rule G must view the site on the day after H views the site.
Answer choice (D) is incorrect because from the fourth rule, when K views the site on day 4, L must view the site on day 5.
Answer choice (E) is the correct answer.
Dave Killoran
PowerScore Test Preparation
Follow me on X/Twitter at http://twitter.com/DaveKilloran
My LSAT Articles: http://blog.powerscore.com/lsat/author/dave-killoran
PowerScore Podcast: http://www.powerscore.com/lsat/podcast/
PowerScore Test Preparation
Follow me on X/Twitter at http://twitter.com/DaveKilloran
My LSAT Articles: http://blog.powerscore.com/lsat/author/dave-killoran
PowerScore Podcast: http://www.powerscore.com/lsat/podcast/